humble tâtonnement sur Fermat Wiles

bonjour,

Par un algorithme d’approximations successives, j’ai trouvé des solutions pour le système suivant :

( y+1)^4 – y^4 – x^4 = 0 avec y appartient à R et x appartient à N

les solutions sont :
x y
46 103,329418
108 323,499743
211 790,828035
365 1642,75064
579 3039,57817
864 5183,49998
1230 8301,57816
1688 12660,7502
Etc …

Soit :
x est à peu près = 1,688 *( k^3) avec k > 2 appartient à N

y est à peu près x*0,75*k avec k > 2 appartient à N

C’est peut-être trivial comme solution mais j’aurai bien aimé comprendre les constantes 1,688 et 0,75

merci

Réponses

  • Bonjour.

    Aucun rapport avec le théorème de Fermat-Wiles (qui parle d'entiers).
    Donc finalement, tu as trouvé des solutions à une équation de degré 3 d'inconnue y pour certaines valeurs du paramètre x. Mais on sait qu'une équation de degré 3 a toujours au moins une solution, et en trouver des valeurs approchées de la ou les solutions est un problème réglé depuis des siècles.

    Pour ta question finale, l'équation s'écrit $ 4y^3+6y^2+4y+1=x^4$ et donc, pour x très grand, y est lui aussi très grand, et les termes autres que le $4y^3$ deviennent négligeables. Donc $4y^3\approx x^4$. Ce qui donne, en posant $x=a\times k^3$ où $k$ est un réel et $a$ a la valeur que tu veux (1,688 si tu veux) :
    $4y^3\approx x\times x^3=a\times k^3\times x^3$
    $y^3\approx \frac a 4 \times k^3\times x^3$
    $y\approx \sqrt[3]{\frac a 4 } \times k\times x$
    Pour a=1,688; $\sqrt[3]{\frac a 4 } \approx 0,7500740668$
    je ne sais pas d'où sortent tes valeurs de x, tu ne dis pas comment tu les as trouvées. Mais le lien que tu trouves entre y et x est cohérent avec une approximation de $ 4y^3+6y^2+4y+1$ par $ 4y^3$.

    Cordialement.
  • Merci de ta réponse.
    Je n'ai pas tout compris dans ta réponse et j'y reviendrai quand j'aurai assimilé.

    Pour l'instant, mon équation est (y+1)4 - y4 - x4 = 0 et je dis comment je trouve x.
    Par un algorithme d’approximations successives, j’ai trouvé des solutions pour le système suivant ci-dessus).
    C'est-à-dire que j'ai commencé par x = 1 et j'ai cherché un y qui satisfaisait l'équation. Le zéro trouvé est de l'ordre de 10-6 par calcul littéral (de mon cru).

    Donc je n'ai pas l'impression qu'il y a d'autres x que ceux que j'ai donné sauf à compléter la suite déjà trouvée;

    Dire que ça n'a rien à voir avec [large]F[/large]ermat whiles [large]W[/large]iles est un peu rapide.
    Si on cherche xn + yn = zn avec n =4 et x,y,z=y+1 appartenant à N3
    Soit x4 + y4 = (y+1)4 avec x,y appartenant à N2
    on montre qu'il n'existe pas de y appartenant à N quel que soit x.
    À plus quand j'aurai mieux assimilé ta démonstration.

    [Pierre de Fermat (1601-1665) et Andrew Wiles (1953-.) prennent toujours une majuscule. AD]
  • Radinor: tu veux une solution à l'"équation de Fermat"?

    essaie: $(x,y,\sqrt[n]{x^n+y^n})$ qui est solution de $x^n+y^n=z^n$

    Oui, en effet, c'est une solution non entière mais il me semble que cela ne te pose pas de problème puisque toi aussi tu utilises des réels qui ne sont pas entiers.
  • Radinor a écrit:
    Donc je n'ai pas l'impression qu'il y a d'autres x que ceux que j'ai donné sauf à compléter la suite déjà trouvée;
    Toute valeur entière de x convient. par exemple x=1 et y=0; x=2 et $y\approx 1.034924112$; x=3 et $y\approx 2.195108785$; etc.
    Plus gênant, pour x=1, la solution exacte y=0 est évidente !

    Et comme tu prends y non entier et z=y+1, ça n'a rien à voir avec la proposition de Fermat. Seule une vague ressemblance reste, mais invoquer un des grands défis des mathématiques des 3 derniers siècles pour parler d'une petite équation dont tu ne vois même pas qu'elle a des solutions pour tout x (entier ou non) c'est seulement publicitaire !

    Désolé, mais tu devrais apprendre suffisamment de mathématiques pour pouvoir comprendre ce qui se passe (disons un niveau fin de lycée, assimilé).
  • bonsoir Fin de partie,

    Ma démarche n'est pas de trouver une solution à Fermat Wiles, au contraire, mais je trouve que passer par les formes modulaires, si c'est artistique et juste, est luxueux.
    je cherche donc une façon plus simple de démontrer qu'il n'y a pas de solution.

    donc montrer qu'il n'y en a pas pour x4 + y4 = (y+1)4
    puis de montrer qu'il n'y en a pas pour x4 + y4 = (y+2)4
    puis finalement qu'il n'y en a pas pour x4 + y4 = (y+p)4

    espérant trouver une piste qui me permet de dire qu'il n'y en pas quelque soit xn + yn = zn

    mais si je ne peux généraliser de suite mais si je trouve quelques pistes je continue.
    je m'arrête si je n'ai pas trouvé de pistes avec x4 + y4 = (y+p)4

    d'où savoir que représente 1,688 et 0,75 serait une piste à moins que ce soit trivial comme l'a dit GérardO
  • Bonsoir GérardO,

    Généralement quand on étudie xn + yn = zn avec x,y,z,n>2 appartient à N[large][/large]4
    on cherche avec z > y > x pour ne pas refaire plusieurs fois les mêmes recherches
    on élimine aussi les kx,ky,kz
    etc ...
    Ce qui fait que les solutions que tu donnes doivent être x=0 , y=1 et x=1.034924112 , y=2 et x=2.195108785 , y=3; etc n'ont pas de sens.

    D'autre part t'es pas tellement sympa avec moi.

    je te ferai signe quand j'aurai trouvé.
  • Radinor,

    je ne cherche pas à être sympa, juste à traiter ta question (j'ai toujours repris ce que tu disais au premier message; et j'ai pris le temps de justifier ton "environ 0,75", ce que tu n'étais pas capable de faire). Et je n'ai toujours pas compris pourquoi tu trouvais seulement certaines valeurs de x et pas les autres. Tu n'as jamais expliqué ça. Maintenant, tu m'eng .. sous prétexte que "quand on étudie xn + yn = zn avec x,y,z,n>2 appartient à N4
    on cherche avec z > y > x pour ne pas refaire plusieurs fois les mêmes recherches "
    Là tu dérives grave, car justement, tu as choisi de ne pas traiter ce problème (ton y n'est plus un entier !!! ). Et tu n'as jamais indiqué cette condition jusque là.

    J'en déduis que tu manques de l'élémentaire politesse de dire de quoi il s'agit, et je te laisse à tes lubies.
Connectez-vous ou Inscrivez-vous pour répondre.